Septaphobia

Calculus Level 3

1 7 1 ! + 1 7 + 2 7 2 ! + 1 7 + 2 7 + 3 7 3 ! + 1 7 + 2 7 + 3 7 + 4 7 4 ! + \frac {1^7}{1!} + \frac {1^7 + 2^7}{2!} + \frac {1^7 + 2^7 + 3^7}{3!} + \frac {1^7 + 2^7 + 3^7 + 4^7}{4!} + \ldots

If the series above equals to W W , what is the value of 24 e × W \frac {24}{e} \times W ?

Note: e = lim n ( 1 + 1 n ) n e = \displaystyle \lim_{n \to \infty} \left (1 + \frac 1 n \right )^n

Inspired by Caleb Townsend

Image Credit: Flickr Gingertwist


The answer is 25685.

This section requires Javascript.
You are seeing this because something didn't load right. We suggest you, (a) try refreshing the page, (b) enabling javascript if it is disabled on your browser and, finally, (c) loading the non-javascript version of this page . We're sorry about the hassle.

2 solutions

Caleb Townsend
Apr 9, 2015

Please see this note for a simpler problem if you have not already seen it. The method used to solve here is the same as that in the note but much lengthier, as a system of 8 8 equations is required, rather than 3. 3. The full solution is still posted below.

W = 1 7 1 ! + 1 7 + 2 7 2 ! + . . . = n = 1 [ k = 1 n k 7 n ! ] = n = 1 [ 1 24 n 2 ( n + 1 ) 2 ( 3 n 4 + 6 n 3 n 2 4 n + 2 ) n ! ] = 1 24 [ n = 1 n ( n + 1 ) 2 ( 3 n 4 + 6 n 3 n 2 4 n + 2 ) ( n 1 ) ! ] W = \frac{1^7}{1!} + \frac{1^7 + 2^7}{2!} + ... \\ = \sum_{n=1}^\infty [\frac{\sum_{k=1}^n k^7}{n!}] \\ = \sum_{n=1}^\infty [\frac{\frac{1}{24}n^2(n+1)^2(3n^4 + 6n^3 - n^2 - 4n + 2)}{n!}] \\ = \frac{1}{24}[\sum_{n=1}^\infty \frac{n(n+1)^2(3n^4 + 6n^3 - n^2 - 4n + 2)}{(n-1)!}] Let S = 24 × W , S = 24\times W, then S = n = 1 n ( n + 1 ) 2 ( 3 n 4 + 6 n 3 n 2 4 n + 2 ) ( n 1 ) ! = n = 0 ( n + 1 ) ( n + 2 ) 2 ( 3 n 4 + 18 n 3 + 35 n 2 + 24 n + 6 ) n ! S = \sum_{n=1}^\infty \frac{n(n+1)^2(3n^4 + 6n^3 - n^2 - 4n + 2)}{(n-1)!} \\ = \sum_{n=0}^\infty \frac{(n+1)(n+2)^2(3n^4 + 18n^3 + 35n^2 + 24n + 6)}{n!} Now define f ( x ) = n = 0 ( n + 1 ) ( n + 2 ) 2 ( 3 n 4 + 18 n 3 + 35 n 2 + 24 n + 6 ) n ! x n f(x) = \sum_{n=0}^\infty \frac{(n+1)(n+2)^2(3n^4 + 18n^3 + 35n^2 + 24n + 6)}{n!}x^n which is the Taylor series of g ( x ) = e x ( 3 x 7 + 96 x 6 + 1064 x 5 + 5040 x 4 + 10206 x 3 + 7728 x 2 + 1524 x + 24 ) . g(x) = e^x(3x^7 + 96x^6 + 1064x^5 + 5040x^4 + 10206x^3 + 7728x^2 + 1524x + 24). Substitute x = 1 x = 1 and the result is S = e ( 3 + 96 + 1064 + 5040 + 10206 + 7728 + 1524 + 24 ) = 25685 e S = e(3 + 96 + 1064 + 5040 + 10206 + 7728 + 1524 + 24) \\ = 25685e and S e = 25685 . \boxed{\frac{S}{e} = 25685}.

Proof that the Taylor series about x = 0 x=0 of g ( x ) g(x) is f ( x ) . f(x).
The numerator of the terms of f ( x ) f(x) is a polynomial of degree 7 , 7, and on account of x n n ! , \frac{x^n}{n!}, the function whose Maclaurin series is f ( x ) f(x) is of the form g ( x ) = e x ( a 1 x 7 + a 2 x 6 + a 3 x 5 + . . . + a 7 x + a 8 ) . g(x) = e^x(a_1x^7 + a_2x^6 + a_3x^5 + ... + a_7x + a_8). The general Maclaurin series of that form is a 8 + ( a 7 + a 8 ) x + ( a 6 + a 7 + a 8 2 ) x 2 + O ( x 3 ) a_8 + (a_7 + a_8)x + (a_6 + a_7 + \frac{a_8}{2})x^2 + O(x^3) Compare this to the first several values of ( n + 1 ) ( n + 2 ) 2 ( 3 n 4 + 18 n 3 + 35 n 2 + 24 n + 6 ) n ! x n \frac{(n+1)(n+2)^2(3n^4 + 18n^3 + 35n^2 + 24n + 6)}{n!}x^n which are 24 , 1548 x , 9264 x 2 , . . . 24,\ 1548x,\ 9264x^2,\ ... a 8 = 24 a 7 = 1548 24 = 1524 a 6 = 9264 1524 12 = 7728 a 5 = 18700 7728 762 4 = 10206 . . . a_8 = 24 \\ a_7 = 1548 - 24 = 1524 \\ a_6 = 9264 - 1524 - 12 = 7728 \\ a_5 = 18700 - 7728 - 762 - 4 = 10206 \\ ... continuing this method, a 4 = 5040 , a 3 = 1064 , a 2 = 96 , a 1 = 3. a_4 = 5040,\ a_3 = 1064,\ a_2 = 96,\ a_1 = 3.

HAHAHA, you're missing out on the most important part, AGAIN!

Pi Han Goh - 6 years, 2 months ago

Log in to reply

What am I missing?

Caleb Townsend - 6 years, 2 months ago

Log in to reply

Your Taylor Series (proof)

Pi Han Goh - 6 years, 2 months ago

Log in to reply

@Pi Han Goh Well I added a comment to see the method used in the sum of squares note, but I will take the time now to show how I arrived at that expression.

Caleb Townsend - 6 years, 2 months ago

Log in to reply

@Caleb Townsend Thanks to your comment , I feel the strong urge to generalize

1 n 1 ! + 1 n + 2 n 2 ! + 1 n + 2 n + 3 n 3 ! + 1 n + 2 n + 3 n + 4 n 4 ! + \frac {1^n}{1!} + \frac {1^n + 2^n}{2!} + \frac {1^n + 2^n + 3^n}{3!} + \frac {1^n + 2^n + 3^n + 4^n}{4!} + \ldots

Pi Han Goh - 6 years, 2 months ago

Log in to reply

@Pi Han Goh So , is it done yet ? Do post it as a note when u generalize it . Your methods are always gr8 !

A Former Brilliant Member - 6 years, 2 months ago

Did the same! But it was tedious! Isn't there any method to do it like this -

n = 1 n 7 ( e + 1 ) n 7 e ( Γ ( n , 1 ) ( n 1 ) ! ) \displaystyle \sum_{n = 1}^{\infty}{{n}^{7}(e+1) - {n}^{7}e(\frac{\Gamma(n,1)}{(n-1)!})}

One can take the gamma function 'back' into integral form and do the calculations then. Actually, I haven't yet worked out the calculations.

Kartik Sharma - 6 years, 2 months ago
Pi Han Goh
Apr 9, 2015

Like Caleb's solution, and from equation 27 and apply properties of Bell numbers . The series becomes

1 24 n = 1 3 n 8 + 12 n 7 + 14 n 6 7 n 4 + 2 n 2 n ! = e 24 [ 3 B 8 + 12 B 7 + 14 B 6 7 B 4 + 2 B 2 ] \begin{aligned} & & \frac 1 {24} \displaystyle \sum_{n=1}^\infty \frac {3n^8 + 12n^7 + 14n^6 - 7n^4 + 2n^2}{n!} \\ &= & \frac {e}{24} \left [ 3B_8 + 12B_7 + 14B_6 - 7B_4 + 2B_2 \right ] \\ \end{aligned}

Substitute those Bell numbers .

Hence the answer is simply 3 B 8 + 12 B 7 + 14 B 6 7 B 4 + 2 B 2 = 25685 3B_8 + 12B_7 + 14B_6 - 7B_4 + 2B_2 = \boxed{25685}

Saved by the Bell, (number, that is). I was about to proceed as per Caleb's method outlined in the note that inspired this question, then I looked into Bell numbers, (nice hint with the picture, by the way), and realized the same approach as yours. Judging from the length of the OEIS entry and all the references, this sequence of numbers seems to be highly significant.

Brian Charlesworth - 6 years, 2 months ago

I have not seen Dobinski's formula before, I have only scratched the surface of combinatorics. That seems to have been useful in this problem.

Caleb Townsend - 6 years, 2 months ago

Log in to reply

You have a nice solution! Yeah, I totally forgotten about the name of the formula until you cited Bell's number then I recalled everything.

Pi Han Goh - 6 years, 2 months ago

0 pending reports

×

Problem Loading...

Note Loading...

Set Loading...